Pharm Principles Of Pharm W Exp

Approved & Edited by ProProfs Editorial Team
The editorial team at ProProfs Quizzes consists of a select group of subject experts, trivia writers, and quiz masters who have authored over 10,000 quizzes taken by more than 100 million users. This team includes our in-house seasoned quiz moderators and subject matter experts. Our editorial experts, spread across the world, are rigorously trained using our comprehensive guidelines to ensure that you receive the highest quality quizzes.
Learn about Our Editorial Process
| By Chachelly
C
Chachelly
Community Contributor
Quizzes Created: 513 | Total Attempts: 592,897
Questions: 20 | Attempts: 171

SettingsSettingsSettings
Pharm Principles Of Pharm W Exp - Quiz

Questions and Answers
  • 1. 

    A 45-year-old man with a medical history of atrial fibrillation and diabetes was recently diagnosed with bone tuberculosis. Oral medications taken by the patient included: Pyrazinamide ......(base, pKa = 10.0) Warfarin ..............(acid, pKa = 5.0) Isoniazid............ (base, pKa = 3.8) Ethambutol ........(base, pKa = 6.3) Glyburide ............(acid, pKa = 5.3) Which of the listed drugs was most ionized in the patient’s stomach lumen (assuming a pH = 1.5?

    • A.

      Pyrazinamide

    • B.

      Warfarin

    • C.

      Isoniazid

    • D.

      Ethambutol

    • E.

      Glyburide

    Correct Answer
    A. Pyrazinamide
    Explanation
    An ionizable drug is more or less ionized according to the pH of the medium. The
    Henderson-Hasselbalch equation predicts that a weak acid will be more ionized when
    pKa < pH, whereas the reverse holds for a weak base. The amount of the ionized drug will be
    proportional to the magnitude of the difference pH-pKa for an acid, and the difference pKa-pH
    for a base. Therefore the drugs can be ranked as follows from the most ionized to the least
    ionized:
    Pyrazinamide
    Ethambutol
    Isoniazid
    Warfarin
    Glyburide

    Rate this question:

  • 2. 

    A 62-year-old man, recently diagnosed with ventricular tachycardia, started a treatment with amiodarone, 200 mg daily. The drug has a Vd of about 1290 L, and an a t½ of about 25 days. Which of the following was most likely the time needed for amiodarone plasma concentration to reach practically the steady state in this patient?

    • A.

      50 days

    • B.

      100 days

    • C.

      25 days

    • D.

      12 days

    • E.

      150 days

    • F.

      125 days

    Correct Answer
    B. 100 days
    Explanation
    According to the four half-life rule, the steady state (exactly 94% of the steady state) will be
    reached in 100 days.

    Rate this question:

  • 3. 

    The pharmacokinetics of a new a drug was studied in laboratory animals. A 200 mg dose of the drug was given IV and blood samples were drawn and analyzed for drug content. The drug concentrations at various times were obtained and plotted below. Which of the following was most likely the Vd of the drug, in liters?

    • A.

      3

    • B.

      5

    • C.

      10

    • D.

      15

    • E.

      20

    • F.

      8

    Correct Answer
    C. 10
    Explanation
    Vd = (Dose x F) / C0
    C0 =(determined from the graph) = 20 mcg/mL
    Vd = 200 mg / 20 mg/L = 10 L

    Rate this question:

  • 4. 

    A 75-year-old woman was treated with doxycycline (200 mg IV every 8 hours) for a severe rickettsial pneumonia. knowing that the clearance of doxycycline is about 22 L/hr, which of the following was most likely the plasma concentration of the drug (in mg/L) at the steady state, in this patient?

    • A.

      0.50

    • B.

      0.05

    • C.

      0.25

    • D.

      1.45

    • E.

      1.13

    • F.

      1.78

    Correct Answer
    E. 1.13
    Explanation
    Css = ?
    Css = DR / CL
    DR = 200 mg IV every 8 hours = 25 mg/hr
    Css =(25 mg/hr) / (22 L/hr) = 1.13 mg/L

    Rate this question:

  • 5. 

    A 54-year-old obese patient (weight: 130 kg), recently diagnosed with essential hypertension, started a treatment with diltiazem, 30 mg tablet twice daily. The drug has a t1/2 of about 4 hours and a Vd of 1.7 L/Kg. Which of the following was most likely the total clearance (in L/hr) of diltiazem in this patient?

    • A.

      45.5

    • B.

      24.2

    • C.

      58.1

    • D.

      38.5

    • E.

      13.4

    • F.

      75.2

    Correct Answer
    D. 38.5
    Explanation
    CL = ?
    CL= 0.7 Vd / t1/2
    Vd = ?
    Vd = 1.7 L/Kg x 130 Kg = 220 L
    CL = 0.7 x 220 L / 4 hr = 38.5 L/hr

    Rate this question:

  • 6. 

    A 17-year-old boy took a tablet of ibuprofen because of a disturbing headache. Ibuprofen is weak acid with a pKa of 4.8. What percentage of the drug was most likely water soluble in the patient’s plasma?

    • A.

      1%

    • B.

      24%

    • C.

      50%

    • D.

      76%

    • E.

      99%

    • F.

      > 99%

    Correct Answer
    F. > 99%
    Explanation
    An acidic drug is mainly water soluble when its pKa is lower than the pH of the medium. The
    amount of water solubility is proportional to the difference between pH and pKa according to
    the following table


    Since the pH of plasma is about 7.4, the drug will be >99% water soluble in the patient’s
    plasma.

    Rate this question:

  • 7. 

    The pharmacokinetics of a new drug that follows a first-order, one compartment model kinetics, was studied in a healthy volunteer. A 20 mg dose was given IV. The plasma concentration of the drug turned out to be 2 mg/L initially, and 1 mg/L 2 hours later. Which of the following was most likely the total clearance of the drug (in L/h)?

    • A.

      2.5

    • B.

      8.4

    • C.

      5.5

    • D.

      6.7

    • E.

      3.5

    • F.

      4.9

    Correct Answer
    E. 3.5
    Explanation
    The Cp of the drug is 2 mg/L initially and 1 mg/L after 2 hours, so the half life of the drug is 2
    hours. Therefore:
    Vd = 20 mg / 2 mg/L = 10 L
    CL = 0.7 Vd/ t½
    CL = 0.7 10 L / 2 h = 3.5 L/h

    Rate this question:

  • 8. 

    A 41-year-old man, admitted to the coronary unit because of a myocardial infarction, developed a sustained tachycardia with atrioventricular dissociation. A loading dose of lidocaine was given IV. Lidocaine has a t½ of 1.8 hours, a total CL of 38 L/hr and is effective at an initial plasma concentration of 3 mg/L. Which loading dose (in mg) was most likely administered to this patient?

    • A.

      294

    • B.

      244

    • C.

      127

    • D.

      98

    • E.

      350

    • F.

      72

    Correct Answer
    A. 294
    Explanation
    Loading dose = (target concentration x Vd) / F
    Vd = ?
    Vd = t½ CL / 0.7
    Vd =1.8 hr x 38 L/hr / 0.7 = 98 L
    Loading dose = 3 mg/L x 98 L = 294 mg

    Rate this question:

  • 9. 

    The pharmacokinetics of a new a drug was studied in a healthy volunteer. A 32mg dose was given IV and blood samples were drawn and analyzed for drug content. The drug concentrations at various times were obtained and plotted below. Which of the following was most likely the total CL of the drug, in L/hr?

    • A.

      2.25

    • B.

      1.40

    • C.

      3.02

    • D.

      2.01

    • E.

      2.87

    Correct Answer
    B. 1.40
    Explanation
    CL = ?
    CL = 0.7 Vd / t1/2
    VD = ?
    Vd = D / C0 = 32 mg / 8 mg/L = 4L
    t1/2 = ?
    t1/2 = (determined from the graph) = 2 hours
    CL = (0.7 x 4 L) / 2 hr = 1.4 L/hr

    Rate this question:

  • 10. 

    The pharmacokinetics of a new drug was studied in a healthy volunteer. The drug was given by IV infusion at a rate of 300 mg/h . The drug concentrations at various times were obtained and the curve interpolating these concentrations is depicted below. Which of the following was most likely the total CL of the drug in L/h?

    • A.

      10

    • B.

      5

    • C.

      20

    • D.

      8

    • E.

      26

    • F.

      6

    Correct Answer
    A. 10
    Explanation
    CL = ?
    CL = DR / Css
    CL= (300 mg/hr)/ (30 mg/L) = 10 L/hr

    Rate this question:

  • 11. 

    The pharmacokinetics of a new drug that follows first-order, one compartment model kinetics, was studied in a healthy volunteer. A 900 mg dose was given by oral route. It was found that the drug had an oral bioavailability of 0.05, volume of distribution (Vd) of about 2500 liters and total clearance of 10 mL/min. Which of the following statements best explains why the drug had such a large Vd in this subject?

    • A.

      It was concentrated intracellularly

    • B.

      It distributed in the total body water

    • C.

      Its administered dose was very high

    • D.

      It had a low oral bioavailability

    • E.

      It had a low total clearance

    Correct Answer
    A. It was concentrated intracellularly
    Explanation
    If a drug concentrates intracellularly it leaves the extracellular compartment and enters the
    cells. Therefore if the Vd of a drug is higher than 42 L (the total body water) we can conclude
    that the drug is concentrated in some cells of the body.
    B) If the drug is distributed in the total body water its Vd would be about 42 liters.
    C) The volume of distribution is not dependent on the administered dose. In fact since Vd =
    Dose/Cp, it is clear that if the dose is increased, the plasma concentration will be increased
    accordingly, and the Vd will stay the same.
    D, E) Absorption (i.e. bioavailability) and elimination (i.e. clearance) have nothing to do with
    distribution.

    Rate this question:

  • 12. 

    A 54-year-old woman, suffering from metastatic breast cancer, started a treatment with buprenorphine by sublingual route. Buprenorphine is a basic drug with a pKa of about 8.4. The sublingual bioavailability of the drug is about 50% whereas its oral bioavailability is < 20%. Which of the following statements best explains the reason of this difference?

    • A.

      Sublingual mucosa has higher absorption capacity than intestinal mucosa

    • B.

      The drug is mainly non-ionized in the intestinal lumen

    • C.

      Saliva pH is higher than the pH of the intestinal lumen

    • D.

      The hepatic clearance of the drug is very high

    • E.

      Surface area of sublingual mucosa is quite large

    Correct Answer
    D. The hepatic clearance of the drug is very high
    Explanation
    Oral bioavailability of a drug is low when its first pass effect is large. On the contrary sublingual
    bioavailability is not affected by the first pass effect since venous drainage from the mouth is to
    the superior vena cava, bypassing the portal circulation.
    A) Intestinal mucosa is the mucosa with the highest absorption capacity.
    B) If the drug is mainly nonionized in the intestinal lumen it can cross the intestinal mucosa
    easily but this is not a sufficient condition for a high bioavailability because of the possibility of
    a large first pass effect.
    C) The pH of the medium can affect the permeation of a drug but is not the only factor that
    regulates the bioavailability.
    E) the surface area of oral muscosa is much smaller than that of the small intestine.

    Rate this question:

  • 13. 

    A 63-year-old woman with atrial flutter had been receiving diltiazem, an antiarrhythmic drug biotransformed by CYP3A4 isozyme. Few days ago she was diagnosed with tuberculosis and started a treatment which included rifampin. Knowing that rifampin can affect diltiazem metabolism, which of the following might occur?

    • A.

      Plasma levels of diltiazem may increase

    • B.

      The pharmacological effects of diltiazem may be reduced

    • C.

      The therapeutic index of diltiazem may decrease

    • D.

      The dose of diltiazem may need to be decreased

    • E.

      Overdose toxicity of diltiazem is more likely

    Correct Answer
    B. The pharmacological effects of diltiazem may be reduced
    Explanation
    Rifampin is a potent inducer of the CYP3A4 isozyme, which is responsible for the metabolism
    of 50% of all drugs. Since diltiazem is biotransformed by that isozyme, more drug will be
    biotransformed and the pharmacological effects of diltiazem will be reduced
    A, D, E) Actually plasma levels of diltiazem are decreased, The dose of diltiazem needs to be
    increased, and overdose toxicity of diltiazem will be less likely.
    C) The therapeutic index of a drug is the ratio between the toxic and the therapeutic dose.
    Since more drug is metabolized, the therapeutic index of the drug should not change, as more
    drug is needed to give both the therapeutic and the toxic effects.

    Rate this question:

  • 14. 

    A 57-year-old man, who was in the hospital after a surgical operation, complained from a severe abdominal pain and the physician decided to start an analgesic treatment. The drugs he was considering were morphine (10 mg IM) and buprenorphine (0.3 mg IM). Morphine is a full agonist at opioid mu receptors whereas buprenorphine is a partial agonist at the same receptors. The above mentioned doses of the two drugs are equieffective doses. Which of the following pairs of statements correctly defines the potency and efficacy of morphine and buprenorphine?

    • A.

      Morphine most potent. Buprenorphine most effective

    • B.

      Morphine most potent. Buprenorphine least effective

    • C.

      Morphine least potent. Buprenorphine most effective

    • D.

      Morphine least potent. Buprenorphine least effective

    Correct Answer
    D. Morphine least potent. Buprenorphine least effective
    Explanation
    Potency of a drug refers to the dose of that drug needed to obtain a given effect. Since 10 mg
    of morphine are needed to get an analgesic effect equal to that given by 0.3 mg of
    buprenorphine, morphine is less potent than buprenorphine.
    Efficacy refer to the maximal effect produced by a drug. By definition partial agonists have a
    maximal efficacy lower than that of full agonists. Since morphine is a full agonist and
    buprenorphine a partial agonist at the same receptor, buprenorphine is less effective than
    morphine.

    Rate this question:

  • 15. 

    Drug A and Drug B, tested in an in vitro system, turn out to have equal affinity for the same receptor and intrinsic activities greater than zero. The maximal efficacy of drug B is 100%. When Drug B is co-administered with a fixed dose of Drug A, the log dose-effect curve of Drug B is shifted to the left.  Which of the following terms best defines drug A?

    • A.

      Full agonist

    • B.

      Partial agonist

    • C.

      Competitive antagonist

    • D.

      Irreversible antagonist

    • E.

      Functional antagonist

    Correct Answer
    A. Full agonist
    Explanation
    Since the two drugs have an intrinsic activity greater than zero they must be either full agonists
    or partial agonists. The maximal efficacy of drug B is 100%, so drug B must be a full agonist.
    Since the log-dose effect curve of B is shifted to the left in the presence of drug A, drug A also
    must be a full agonist. In fact lower doses of a full agonist are needed to get the same
    response in the presence of another full agonist, since some receptors are already activated
    by the other agonist.
    B) As a rule, a partial agonist behaves as a competitive antagonist when given together with a
    full agonist. Therefore if drug A were a partial agonist the dose-effect curve of drug B would
    have been shifted to the right, not to the left.
    C, D) By definition competitive and irreversible antagonists are devoid of intrinsic activity
    E) A functional antagonist acts on receptors different from those activated by the agonist

    Rate this question:

  • 16. 

    The graph below represents the relationship between increasing doses of a drug and percent individuals showing a predefined effect. Which of the following phrases best defines the depicted relationship?

    • A.

      Graded log dose-response curve

    • B.

      Cumulative frequency distribution of doses

    • C.

      Therapeutic/ toxic dose relationship

    • D.

      Graded dose-response curve

    • E.

      Quantal log dose-response curve

    Correct Answer
    E. Quantal log dose-response curve
    Explanation
    The figure depicts a log-dose response relationship (on the x-axis the doses are in a log
    scale). Log-dose response relationship can be:
    a) Graded, when the ordinate indicates the magnitude of a given effect
    b) Quantal, when the ordinate indicates the percent of individuals showing a predefined effect.
    Therefore the quantal curve depicts a cumulative frequency distribution of responders versus
    the log dose.
    A, D) (see explanation above)
    B) The figure represents the cumulative frequency distribution of responders, not of doses
    C) The relationship between the therapeutic and the toxic dose of a drug is given by the
    therapeutic index or the therapeutic window, not by a single log-dose response curve.

    Rate this question:

  • 17. 

    In the figure below, curve X depicts the log dose-response curve for a full alpha-1 agonist on contraction of vascular smooth muscle. Which of the curves best depicts the log dose-response curve of that agonist when a fixed dose of a competitive alpha-1 antagonist is given concomitantly?

    • A.

      Curve A

    • B.

      Curve B

    • C.

      Curve C

    • D.

      Curve D

    • E.

      Curve E

    Correct Answer
    E. Curve E
    Explanation
    When a dose of a competitive antagonist is given in the presence of a dose of a full agonist the
    affinity of the agonist for its receptor is decreased since some receptors will be occupied by the
    antagonist. Therefore the dose response curve of an agonist in the presence of a fixed dose of
    a competitive antagonist will be shifted to the right, but the maximal response will not be
    affected since the competitive antagonism is surmountable. In other words all the receptors
    can be occupied by the agonists even in the presence of a competitive antagonist, provided
    that the amount of the agonist is high enough (remember that the interaction between drug and
    receptors is driven by the law of the mass action).

    Rate this question:

  • 18. 

    A 65-year-old woman, admitted to the hospital with a myocardial infarction, developed a ventricular tachycardia and received an IV injection of lidocaine. The cardiologist knew that the dose given must be within the range of doses that have a high probability of therapeutic success. Which of the following terms best defines this range?

    • A.

      Intrinsic activity

    • B.

      Efficacy

    • C.

      Potency

    • D.

      Therapeutic index

    • E.

      Therapeutic window

    • F.

      Response variability

    Correct Answer
    E. Therapeutic window
    Explanation
    The interval between the minimum therapeutic dose (or plasma concentration) and the
    minimum toxic dose (or plasma concentration) of a drug is defined therapeutic window. Since
    doses below the minimum therapeutic dose are by definition ineffective, and doses above the
    minimum toxic dose are, by definition, toxic, this window defines the range of doses which have
    a high probability of therapeutic success.
    A) Intrinsic activity refers to the ability of a drug (once bound to a receptor) to initiate changes
    which lead to a biological response.
    B) Potency refers to the dose of a drug required to produce a given effect
    C) Efficacy refers to the maximal effect that a drug. can produce
    D) The therapeutic index is a ratio between an harmful dose and an effective dose of a drug . It
    is (like the therapeutic window) an index of the safety of a drug, but is a single value and
    therefore cannot predict the range of doses that are both safe and effective.
    F) The definition refers to the range of doses not to the range of responses.

    Rate this question:

  • 19. 

    A 17-year-old girl started smoking one month ago and now she is smoking 5-10 cigarettes daily. She has noticed that the first cigarette of the day often causes mild tachycardia, which usually disappears with the following cigarettes. Which of the following terms best defines this tolerance pattern?

    • A.

      Cross tolerance

    • B.

      Tachyphylaxis

    • C.

      Pharmacokinetic tolerance

    • D.

      Innate tolerance

    • E.

      Sensitization

    Correct Answer
    B. Tachyphylaxis
    Explanation
    Tachyphylaxis refers to a drug tolerance that appears rapidly (in a matter of hours) and
    disappears also rapidly when the drug is withdrawn. So the first cigarette caused tachycardia
    since tolerance to nicotine effects disappeared overnight, but tolerance to that effect was
    rapidly resumed when smoking was reinstated.
    A) Cross tolerance refers to tolerance to a drug that is shared also by other drugs with similar
    chemical structure and /or similar pharmacological effects.
    C) Tolerance is called pharmacokinetic when is due to a decrease in the effective
    concentration of the drug at the site of action.
    D) Innate tolerance refers to a genetically determined lack of sensitivity to a drug that is
    observed the first time the drug is administered.
    E) Sensitization refers to a drug response that increases with the repetition of the same dose
    of that drug.

    Rate this question:

  • 20. 

    A 54-year-old man had been diagnosed with peptic ulcer and an appropriate therapy was prescribed. The therapy included antacids ‘as needed’, in order to antagonize the effects of the hydrochloric acid secreted by gastric parietal cells. Which of the following terms best defines this acid-antacid antagonism?

    • A.

      Competitive

    • B.

      Noncompetitive

    • C.

      Functional

    • D.

      Chemical

    • E.

      Pharmacokinetic

    Correct Answer
    D. Chemical
    Explanation
    Antacid drugs are weak bases that react chemically with gastric hydrochloric acid to form
    a salt and water. In this way gastric acidity is reduced. This acid-antacid antagonism does not
    involve pharmacokinetic action or receptor occupancy but only a chemical combination
    between two molecules. It is therefore defined chemical antagonism.
    A, B, C) All these antagonism involve receptor occupancy.
    E) Pharmacokinetic antagonism occurs when a drug prevent the absorption or stimulates the
    elimination of the drug to be antagonized, so reducing its concentration at the site of action.

    Rate this question:

Quiz Review Timeline +

Our quizzes are rigorously reviewed, monitored and continuously updated by our expert board to maintain accuracy, relevance, and timeliness.

  • Current Version
  • Mar 20, 2023
    Quiz Edited by
    ProProfs Editorial Team
  • Jun 04, 2012
    Quiz Created by
    Chachelly
Back to Top Back to top
Advertisement
×

Wait!
Here's an interesting quiz for you.

We have other quizzes matching your interest.